0
$\begingroup$

There is a theorem by Mooney http://msp.org/pjm/1972/43-2/pjm-v43-n2-p.pdf#page=185 and independently proved by Havin which says that the predual of $H^{\infty}(\mathbb{D}),$ $L^{1}/H^{1}_{0}$ is weakly sequentially complete, i.e., to say if we have a sequence of weak* continuous linear functionals $\lbrace\Lambda_n;n\in\mathbb{Z}_{+}\rbrace$ such that $\Lambda\varphi:=\lim\Lambda_n\varphi$ exists for each $\varphi\in H^{\infty}(\mathbb{D}).$ Then $\Lambda$ is weak* continuous. I have following questions:

  1. is the conclusion still holds true if the sequential limit exists apriori only for $\varphi\in A(\mathbb{D})?$

  2. or if there are some results about the maximal set where if the sequential limit exits the conclusion of the above theorem still holds?

Remark: $H^{\infty}(\mathbb{D})$ and $A(\mathbb{D})$ respectively represents the Banach spaces of bounded analytic functions and the holomorphic functions which are continuous upto the boundary of $\mathbb{D}.$

$\endgroup$

1 Answer 1

2
$\begingroup$

I don't about the second question, but the answer to the first question is "no," since one can take a sequence $\Lambda_n$ of $L^1$ functions converging (weak*, in the dual of $C(\mathbb T)$) to the point mass at 1, say. Then take $\Lambda$ to be a Hahn-Banach extension to $H^\infty$ of the evaluation functional $\varphi\to \varphi(1)$ on $A(\mathbb D)$. We then have $\Lambda_n\to \Lambda$ pointwise on $A(\mathbb D)$ but $\Lambda$ is not weak* continuous.

$\endgroup$

Your Answer

By clicking “Post Your Answer”, you agree to our terms of service and acknowledge you have read our privacy policy.

Not the answer you're looking for? Browse other questions tagged or ask your own question.